Dériver l'équation d'Euler


7

Je veux dériver l'équation d'Euler pour les éléments suivants:

$$ max \ sum \ limits_ {t = 0} ^ {T} = \ beta ^ {t} U (C_t) $$

$$ s.t. C_t + K_ {t + 1} \ leq f (K_t), t = 0,1,2, ..., T-1 $$ $$ - K_ {T + 1} \ leq 0 $$

Je suis un peu confus quant à la raison pour laquelle le FOC avoir cela:

$$ \ frac {d \ mathcal {L}} {dK_ {t + 1}} = - \ lambda_t + \ lambda_ {t + 1} f '(k_ {t + 1}) $$

et comment nous combinons le F.O.C pour donner l'équation d'Euler:

$$ U '(C_t) = \ beta U' (C_ {t + 1}) f '(k_ {t + 1}) $$

Je suppose que le F.O.C. $ K_ {t + 1} $ est tel à cause de l'inclusion de la forme intensive de la fonction de production mais je ne sais pas exactement comment et je veux vraiment comprendre cela complètement. Je dois également m'assurer de bien comprendre comment nous utilisons le FOC pour produire l'équation d'Euler. Quelqu'un peut-il fournir un peu de clarté?

Réponses:


3

La question est assez simple, et vous n'avez pas besoin de la première étape que vous avez. Vous avez (pour une raison quelconque) un multiplicateur différent ici pour chaque période. Ce n'est pas le cas, vous avez simplement un condition de schéma no-ponzi, également appelée contrainte de transversalité. Le problème est exprimé sous la forme $ max \, \ sum \ beta ^ {t} U (c_ {t}) $ st $ c_ {t} + k_ {t + 1} \ leq f (k_ {t}) $. Le flux d’utilité pour l’agent est $ U = \ sum \ beta ^ {t} U (Cc_ {t}) = U (c_ {0}) + .... \ beta ^ {t} U (c_ {t }) + \ beta ^ {t + 1} U (c_ {t + 1}) + ... + \ beta ^ {T} U (c_ {T}). $

Substituez dans $ f (k_ {t + 1}) - k_ {t} $ pour $ c_ {t} $. Ensuite, nous en déduisons $ k_ {t + 1} $ et égal à 0: $ \ frac {\ U partiel} {\ partiel k_ {t + 1}} = \ beta ^ {t} U '(c_ {t}) - \ beta ^ {t + 1} U '(c_ {t + 1}) f' (k_ {t + 1}) = 0 \ Rightarrow U '(c_ {t}) = \ beta U' (c_ {t + 1} ) f '(k_ {t + 1}). $

Vous n'avez pas besoin de l'autre équation dont vous avez parlé.


Oh oui. Désolé ... je sens que c'est une question vraiment stupide maintenant. Je viens de substituer ceci dans et élargi et vois tout très clairement. Merci pour la réponse.
123

1
On ne peut pas faire la substitution que vous suggérez tant que la relation est une inégalité. Il faut d'abord expliquer pourquoi cela sera considéré comme une égalité.
Alecos Papadopoulos

2
@ChinG En effet, et c'est l'argument qui transforme l'inégalité en une égalité, et il a donc appartenu au corps principal de votre réponse, c'était le point de mon commentaire. Parce que le problème a une solution même si nous maintenons l'inégalité (dans ce cas, il faut utiliser des multiplicateurs).
Alecos Papadopoulos

1
@Alecos Merci! Je vais essayer d'être plus complet à l'avenir.
ChinG

1
@AlecosPapadopoulos - Oui, c'est vrai. Cependant, j'ai supposé qu'il prenait l'argument comme une évidence. Merci aussi pour votre réponse. Les deux ensemble ont fourni beaucoup de clarté.
123

2

Le problème est complet $$ \ max _ {\ {C_t, K_ {t + 1} \} _ 0 ^ {\ infty}} \ somme \ {t = 0} ^ {T} \ bêta {{}} U (C_t) $$

$$ s.t. \; \; C_t + K_ {t + 1} \ leq f (K_t), t = 0,1,2, ..., T-1, \; \; \; \; \; -K_ {T + 1} \ leq 0 $$

Donc, nous maximisons aussi en ce qui concerne consommation . Le lagrangien est

$$ \ mathcal L = \ sum \ limits_ {t = 0} ^ {T} \ Big (\ beta ^ {t} \ big [U (C_t) + \ lambda_t \ big (f (K_t) - C_t-K_ { t + 1} \ big) \ big] \ big) $$

Notez que le facteur d'actualisation réduit également la contrainte. ensuite

$$ \ frac {d \ mathcal {L}} {dC_t} = \ beta ^ {t} \ big (U '(C_t) - \ lambda_t \ big) = 0 \ implique U' (C_t) = \ lambda_t $$

et ainsi aussi $ U '(C_ {t + 1}) = \ lambda_ {t + 1} $

En outre,

$$ \ frac {d \ mathcal {L}} {dK_ {t + 1}} = - \ beta ^ t \ lambda_t + \ beta ^ {t + 1} \ lambda_ {t + 1} f '(k_ {t + 1}) = 0 \ implique - \ lambda_t + \ beta \ lambda_ {t + 1} f '(k_ {t + 1}) = 0 $$

En combinant et en réorganisant, on obtient l'équation d'Euler.


1

Je ne suis pas sûr de bien comprendre votre question, mais je vais essayer.

Si vous êtes confus au sujet du premier FOC que vous avez écrit (cela semble correct): Vous n’avez plus seulement une contrainte, comme vous êtes peut-être habitué aux cours de base en économie. Vous avez T contraintes, la contrainte une fois pour chaque période. Donc vous avez t multiplicateurs de Lagrange Lambda. Ecrivez la contrainte par exemple pour trois t et vous verrez ce que je veux dire. Votre contrainte est quelque chose comme lammbda_t * (C_t + Kt + 1 - f (Kt)) + lambda_t + 1 * (C_t + 1 + Kt + 2 - f (Kt + 1)) + lambda_t + 2 * (C_t + 2 + Kt + 3 - f (Kt + 2)) et ainsi de suite jusqu'à T.

Maintenant, pour obtenir l'équation d'Euler: Si vous prenez la dérivée de cela en ce qui concerne K_t + 1, vous obtiendrez votre FOC. (C'est le FOC pour l'ensemble du lagrangien, car la dérivée de U (C) par rapport à K est 0 ici, car toute dépendance de C sur K est déjà dans la contrainte.)

Votre équation d'Euler implique 3 variables inconnues: Ct, Ct + 1 et Kt + 1. Par conséquent, vous aurez besoin de trois FOC. Le maximum no. de FOC que vous avez ici est 2T (T fois pour chaque C et T fois pour chaque K).

Comme vous le voyez, le FOC que vous avez déjà contient 2 choses pour lesquelles vous souhaitez vous débarrasser de lambda_t et de lambda_t + 1. Aussi, vous voulez obtenir les utilitaires marginaux de C_t et C_t + 1. Prenez donc un dérivé par rapport à Ct et Ct + 1 de votre lagrangien. Astuce: L’un d’eux sera: ß ^ t * U '(Ct) - lambda_t = 0.

Rassemblez ces 3 équations maintenant, éliminez les lambdas et vous devriez obtenir votre équation d'Euler.


1

Il n'y a pas de contrainte d'emprunt d'une période à l'autre. Vous avez seulement une condition de transversalité. Cependant, la contrainte sera contraignante si vous avez des conditions Inada, auquel cas vous ne pouvez pas avoir besoin les multiplicateurs.

En utilisant notre site, vous reconnaissez avoir lu et compris notre politique liée aux cookies et notre politique de confidentialité.
Licensed under cc by-sa 3.0 with attribution required.